Đến nội dung

An Infinitesimal nội dung

Có 155 mục bởi An Infinitesimal (Tìm giới hạn từ 06-05-2020)



Sắp theo                Sắp xếp  

#702237 Định lý Cayley - Hamilton (Thắc mắc)

Đã gửi bởi An Infinitesimal on 25-02-2018 - 13:19 trong Đại số tuyến tính, Hình học giải tích

Chứng minh đẳng thức đó mà cũng dùng đến định lý Hamilton Calley thì hơi kỳ cục!




#702393 Định lý Cayley - Hamilton (Thắc mắc)

Đã gửi bởi An Infinitesimal on 27-02-2018 - 18:12 trong Đại số tuyến tính, Hình học giải tích

Riêng quả viết "thay" $\lambda$ bởi $A$ và $1$ bởi $E$ là thấy bá đạo rồi. Chắc lại sách mấy trường kinh tế - kĩ thuật, toàn mấy ông lởm khởm viết. 

Đó là nội dung định lý Hamilton Caylley!




#702667 tính $lim \frac{u_{n+1}-u_{n}}{u...

Đã gửi bởi An Infinitesimal on 03-03-2018 - 16:08 trong Dãy số - Giới hạn

bạn có thể làm chi tiết được không ạ , mình cảm ơn 

 

Vì $u_{n+1}\ge 3 u_n>0, \forall n\in \mathbb{N}$ nên $\lim u_{n}=\infty.$

 

Đặt $f(x)= \sqrt{9x^2+11x+3}.$

 

Khi đó, $\frac{u_{n+1}-u_n}{u_{n+1}+u_n}=\frac{f(u_n)-u_n}{f(u_n)+u_n}.$

 

Từ $\displaystyle\lim_{x\to \infty}\frac{f(x)-x}{f(x)+x}= \lim_{x\to \infty}\frac{\sqrt{9+\frac{11}{x}+\frac{3}{x^2}}-1}{\sqrt{9+\frac{11}{x}+\frac{3}{x^2}}+1}=\frac{1}{2}.$

Suy ra 

$$\displaystyle\lim_{n\to \infty}\frac{u_{n+1}-u_n}{u_{n+1}+u_n}=\frac{1}{2}.$$




#702613 tính $lim \frac{u_{n+1}-u_{n}}{u...

Đã gửi bởi An Infinitesimal on 02-03-2018 - 17:22 trong Dãy số - Giới hạn

cho dãy số $u_{n}$ được xác định bởi $u_{1}= \sqrt{3}$ và $u_{n+1} =\sqrt{9u_{n}^{2} +11u_{n} +3}$

Tính $lim \frac{u_{n+1}-u_{n}}{u_{n+1}+ u_{n}}$

Dùng thông tin $\lim u_n=\infty$ để tính giới hạn cần tìm như giới hạn hàm số!




#701568 Cho dãy số $(u_{n})$ xác định bởi

Đã gửi bởi An Infinitesimal on 12-02-2018 - 21:20 trong Dãy số - Giới hạn

Cho dãy số $(u_{n})$ xác định bởi $\left\{\begin{matrix} u_{n}>0 & & \\ u_{n}^{2}\leq u_{n}-u_{n+1},\forall n\geq 1 & & \end{matrix}\right.$

$a)$ Chứng minh: $u_{n}<\frac{1}{n}, \forall n\geq 1$

$b)$ Tính $limu_{n}$

Ý a) Chứng minh bằng phương pháp qui nạp.

Ý chính $u_{n+1}\le u_n-u_n^2< \frac{1}{n}-\frac{1}{n^2}<\frac{1}{n+1}.$




#701662 Cho dãy số $(u_{n})$ xác định bởi

Đã gửi bởi An Infinitesimal on 14-02-2018 - 16:56 trong Dãy số - Giới hạn

Dạ chỗ này: $u_n-u_n^2< \frac{1}{n}-\frac{1}{n^2}$

 

Tại sao em nghĩ thế? Ta sẽ xem xét điều đó với $n\ge 2, 0<u_n<\frac{1}{n}\le \frac{1}{2}.$

 

Hàm $ g(x)= x-x^2 $ là hàm đồng biến trên $ \left. \left(0,\frac{1}{2}\right.\right].$

 

(Thay vì dùng tính đồng biến, em có thể lập hiệu và phân tích thành nhân tử.)




#702720 $\left\{\begin{matrix} x^{2}+y^{2}=2 & & \...

Đã gửi bởi An Infinitesimal on 04-03-2018 - 07:30 trong Phương trình - hệ phương trình - bất phương trình

Dạ đúng là như vậy huynh ạ ^^

Hình như người ra đề không biết "cộng". Không biết $15+1=?$.

 

 

Từ phương trình thứ 2, ta thu được $y=0 \vee 4y^3+3xy^2-10x=0.$

TH1: $y=0$. Khi đó,  $x=\pm \frac{\sqrt{5}}{2}$.

 

TH2: $4y^3+3xy^2-10x=0$. Kết hợp PT thứ nhất, ta được phương trình đẳng cấp

$4y^3+3xy^2-2x(4x^2+y^2)=0$.

 

Dễ thấy $x\neq 0$. Đặt $t=\frac{y}{x},$ ta thu được phương trình 

$4t^3-2t^2+3t-8=0.$

 

Giải phương trình bậc ba theo cách giải tổng quát, ta thu được

\[t=\frac{\sqrt[3]{12 \sqrt{18633} + 1628}}{12} - \frac{\sqrt[3]{12 \sqrt{18633} - 1628}}{12} + \frac{1}{6}.\]

(Xấu thì làm theo "cách xấu" thôi!)




#701621 Cho dãy số $(u_{n})$ xác định bởi

Đã gửi bởi An Infinitesimal on 13-02-2018 - 20:23 trong Dãy số - Giới hạn

Hình như chỗ này bị ngược dấu thì phải?

 

Viết thế thì ai biết chỗ nào?




#701738 Cho dãy số $(u_{n})$ xác định bởi

Đã gửi bởi An Infinitesimal on 17-02-2018 - 10:01 trong Dãy số - Giới hạn

Ý em là $u_{n}^{2}< \frac{1}{n^{2}}$ $<=> -u_{n}^{2}>-\frac{1}{n^{2}}$ nên em nghĩ bđt trên chưa chắc đúng.

 

Em ĐOÁN sai ý!

Chứng minh trên không phải tiếp cận thông qua tổng BĐT!




#701965 $\left\{\begin{matrix} x^{2}+y^{2}=2 & & \...

Đã gửi bởi An Infinitesimal on 20-02-2018 - 22:44 trong Phương trình - hệ phương trình - bất phương trình

 

xet pt $(2) \iff 2x^2=2+2xy^3 \iff x^2-y^2=2xy^3 (3)$
Nhận vế của pt$(1)$ voi $(3)\iff (x^2-y^2)(x^2+y^2)=4xy^3$
................................................................................................

 

Tiếp cận này cho cần giải PT bậc 4 theo $t=\frac{x}{y}$ "đẹp".

Ngược lại, hướng tiếp cận bên dưới  dẫn đến giải PT bậc 4 khó hơn!

 

 

Đặt $a=x^2, b=y^2$, Hệ không hoàn toàn theo $a, b:$

\begin{cases} \begin{matrix} a+b=2,\\ a-xy b=1. \end{matrix}\end{cases}

Khi đó, $a=\frac{2 xy + 1}{xy + 1}, b=\frac{1}{xy+1}.$

Từ đó, ta dẫn về phương trình theo $xy$:

$(xy)^2=ab= \frac{2 xy + 1}{xy + 1}.\frac{1}{xy+1}.$

Đặt $t=xy,$ ta có

$$ t^4 + 2t^3 + t^2 - 2t - 1=0.$$




#702160 $\left\{\begin{matrix} x^{2}+y^{2}=2 & & \...

Đã gửi bởi An Infinitesimal on 24-02-2018 - 02:08 trong Phương trình - hệ phương trình - bất phương trình

Đề đúng ạ ^^

 

$15y^4+y^4$???




#702030 $\left\{\begin{matrix} x^{2}+y^{2}=2 & & \...

Đã gửi bởi An Infinitesimal on 21-02-2018 - 20:06 trong Phương trình - hệ phương trình - bất phương trình

Cảm ơn huynh đài ^^!
Huynh có thể chỉ cho đệ rằng nếu gặp một số hệ khác đưa về pt t=x/y xấu thì có kinh nghiệm gì & phương pháp gì  để giải không ạ ^^
Ví Dụ đệ gặp bài hệ này  $\left\{\begin{matrix}
4x^2+y^2=5 &  & \\
 15y^4+y^4+12x^2y^2-40xy=0&  &
\end{matrix}\right.$  đệ cũng đưa về đ.c đồng bậc 4 nhưng  k bt làm thế nào nữa @[email protected]
 

 

Có phải em gõ nhầm ở PT thứ 2 không?




#715369 Chứng minh rằng: $n!>(\frac{n}{3})^...

Đã gửi bởi An Infinitesimal on 10-09-2018 - 02:05 trong Dãy số - Giới hạn

có đơn giản quá không nhỉ

 

Ý bạn là thế nào?

 

$e=\lim_{x\rightarrow \infty }(1+\frac{1}{x})^x$

 

Vậy làm sao định nghĩa số $(1+\frac{1}{x})^x$ khi $x$ là số vô tỷ?




#713782 Chứng minh rằng: $n!>(\frac{n}{3})^...

Đã gửi bởi An Infinitesimal on 03-08-2018 - 17:58 trong Dãy số - Giới hạn

Có lẽ bài toán của mình chính là như kiểu đang chứng minh dãy $\left\{\left(1+\frac{1}{n}\right)^n\right\}$ là dãy tăng và hội tụ về $e.$

 

Mình đọc thấy có điều gì đó kỳ kỳ! Vậy e là gì?




#702238 $4U_{n+1}=5U_{n}+3\sqrt{U_{n}^...

Đã gửi bởi An Infinitesimal on 25-02-2018 - 13:22 trong Dãy số - Giới hạn

Cho dãy $(Un)$: $\left\{\begin{matrix}u_{1}=4 & \\ 4U_{n+1}=5U_{n}+3\sqrt{U_{n}^{2}-16} & \end{matrix}\right.$

Tính giới hạn của:$\sum_{n=1}^{2017}=\frac{U_{n}}{2^{2018-n}}$

 

Gõ đề sai rồi!




#702332 $4U_{n+1}=5U_{n}+3\sqrt{U_{n}^...

Đã gửi bởi An Infinitesimal on 26-02-2018 - 18:48 trong Dãy số - Giới hạn

S

 

??? đề đúng thưa anh

Sai ở mức độ nghiêm trọng! 




#702360 $4U_{n+1}=5U_{n}+3\sqrt{U_{n}^...

Đã gửi bởi An Infinitesimal on 26-02-2018 - 22:08 trong Dãy số - Giới hạn

....

Tính giới hạn của:$\sum_{n=1}^{2017}=\frac{U_{n}}{2^{2018-n}}$




#713654 Chứng minh rằng: $n!>(\frac{n}{3})^...

Đã gửi bởi An Infinitesimal on 01-08-2018 - 16:01 trong Dãy số - Giới hạn

 

 

Chứng minh rằng: $n!>(\frac{n}{3})^{n}$ với $\forall n\in\mathbb{N}^*$

 

Bất đẳng thức "mạnh hơn" là $n!>(\frac{n}{e})^{n} \forall n\ge 1.$

 

Đặt $u_n= \dfrac{n!}{\left(\frac{n}{e}\right)^n}.$

 

Ta có $\frac{u_{n+1}}{u_n}=\dfrac{e}{\left(1+\frac{1}{n}\right)^n}>1.$

 

Lưu ý: Khi định nghĩa, $e$, ta đã có dãy $\left\{\left(1+\frac{1}{n}\right)^n\right\}$ là dãy tăng và hội tụ về $e.$

 

Hơn nữa, $u_1>1$ nên $u_n>1 \forall n\ge 1.$ Suy ra điều phải chứng minh.




#715370 $\sum_{n=1}^{+\infty}{\frac...

Đã gửi bởi An Infinitesimal on 10-09-2018 - 02:11 trong Giải tích

Kết hợp sao vậy bạn? Giải thích rõ hơn dùm mình nha! Cảm ơn!

 

1) Khảo sát sự hội tụ của chuỗi số sau:

                    $\sum_{n=1}^{+\infty}{\frac{\cos{n}}{n}}.$

2) Dùng tiêu chuẩn Cauchy xét tính hội tụ của dãy số sau:

$S_n=\frac{\cos{1^n}}{2^1}+\frac{\cos{2^n}}{2^2}+...+\frac{\cos{n^n}}{2^n}.$

 

$T_n=\frac{|\cos{1^n}|}{2^1}+\frac{|\cos{2^n}|}{2^2}+...+\frac{|\cos{n^n}|}{2^n}$. Dãy $\left\{ T_n\right\}$ hội tụ vì dãy này tăng và bị chặn trên bởi 1. 
Tìm ra chặn trên của dãy nhờ đánh giá $\frac{1}{2}+\frac{1}{2^2}+...+\frac{1}{2^n}<1.$

Suy ra $\left\{ S_n\right\}$ hội tụ.




#709110 $u_{0}=\frac{1}{2},u_{k+1}=...

Đã gửi bởi An Infinitesimal on 23-05-2018 - 13:39 trong Dãy số - Giới hạn

Cho dãy $(u_{n})$ thoả mãn: $\left\{\begin{matrix} u_{0}=\frac{1}{2}\\u_{k+1}=u_{k}+\frac{1}{n}u_{k}^{2},\forall k=\overline{0,n-1} \end{matrix}\right.$

Tìm $\lim u_{n}$

 

Đề sai rồi!




#712565 Tìm giới hạn dãy $1+\frac{1}{2} +...+\frac...

Đã gửi bởi An Infinitesimal on 15-07-2018 - 13:23 trong Dãy số - Giới hạn

Tìm giới hạn dãy $1+\frac{1}{2} +...+\frac{1}{n+1}$

P/s:Mong mọi người bỏ ra chút thời gian giúp mình với ạ!

 

Dùng đánh giá $\ln{(1+x)}\le x,$ ta có 

$\frac{1}{k}\ge \ln\left( 1+\frac{1}{k}\right)=\ln{(k+1)}-\ln k. $

Suy ra 

$$1+\frac{1}{2} +...+\frac{1}{n+1}\ge \ln{(n+1).}$$

Do đó, 

$$\lim_{n\to\infty} \left(1+\frac{1}{2} +...+\frac{1}{n+1}\right)=\infty.$$




#709283 $u_{0}=\frac{1}{2},u_{k+1}=...

Đã gửi bởi An Infinitesimal on 26-05-2018 - 14:09 trong Dãy số - Giới hạn

Lời giải:

 

 

Bài này cũng có thể giải được sao? Ngay cả $u_2$, mình cũng không biết xác định như thế nào!




#709324 $u_{0}=\frac{1}{2},u_{k+1}=...

Đã gửi bởi An Infinitesimal on 27-05-2018 - 00:17 trong Dãy số - Giới hạn

Để cho dễ hiểu, cái đề cần phải sửa lại thế này :

Cho $(u_n)$ là một dãy số hữu hạn gồm n+1 số hạng : $u_0,u_1,u_2,...,u_n$ thỏa mãn :

$\left\{\begin{matrix}u_0=\frac{1}{2}\\u_{k+1}=u_k+\frac{1}{n}\ u_k^2,\forall k=\overline{0,n-1} \end{matrix}\right.$

Cho $n$ tiến đến vô cùng, hãy tính $\lim u_n$ ?

 

(Tức là với mỗi giá trị của $n$, ta có một dãy số hữu hạn khác nhau (với số hạng cuối cùng là $u_n$). Cần tính xem khi $n$ tiến đến vô cùng thì số hạng cuối cùng đó tiến đến bao nhiêu ?)

 

Vậy đó là một đề bài khác, không phải đề bài này.




#718226 Xét hội tụ $\int_{0}^{1}\frac{\s...

Đã gửi bởi An Infinitesimal on 08-12-2018 - 02:01 trong Giải tích

Xét sự hội tụ của các tích phân

1. $\int_{0}^{1}\frac{\sqrt[5]{x^{2}+x^{3}}lnx}{x(2-x)}dx$

2. $\int_{1}^{+\infty }\frac{lnx}{2+\sqrt[3]{x^{5}}}dx$

3. $\int_{0}^{1}\frac{sin(\pi x).ln(x-1)}{\sqrt{(x-1)^{3}}}$

 

Mọi người cho e xin phương pháp hay tài liệu giải các dạng này với ạ. E thực sự rất cần !

 

Dùng tiêu chuẩn so sánh "dạng giới hạn" thôi!

 

Bài 3 sai!




#701951 $latex \displaystyle \left( \frac{{{U...

Đã gửi bởi An Infinitesimal on 20-02-2018 - 21:01 trong Dãy số - Giới hạn

Đúng như anh An infinitesimal em giải bài này theo định lý weirstrass cùng với bổ đề Cesaro nhưng mà để ý một chút ta có thể giải bài trên theo nguyên lí ánh xạ co kết hợp với bổ đề Cesaro .

Ta xét hàm: $f(x)=\frac{3n+2}{4n+2}x+\frac{3n+2}{4n+2}$

$\Rightarrow f'(x)=\frac{3n+2}{4n+2}=q<1,  \forall n\in \mathbb{N}^{*}$

Áp dụng định lý Lagrange cho $x,y \in \mathbb{R}, x>y$, $f(x)$ liên tục nên tồn tại $z\in \mathbb{R}$ sao cho:

$f(x)-f(y)=f'(z)(x-y)$

$\left | f(x)-f(y) \right |\leq q\left | x-y \right |$ nên hàm số $f(x)=\frac{3n+2}{4n+2}$ là hàm số co do đó dãy số $(u_{n})$ hội tụ.

 

Đoạn này rối beng nè! Anh sẽ xem xét kỹ hơn sau!

(Dường như em quên đi sự thay đổi của n?)